Difference between revisions of "2023 AMC 12A Problems/Problem 14"

(Solution 1)
Line 9: Line 9:
  
 
~plasta
 
~plasta
 +
 +
==Solution 2==
 +
 +
Let <math>z = re^{i\theta}.</math> We now have <math>\overline{z} = re^{-i\theta},</math> and want to solve
 +
 +
<cmath>re^{5i\theta} = re^{-i\theta}.</cmath>
 +
 +
From this, we have <math>r = 0</math> as a solution, which gives <math>z = 0</math>. If <math>r\neq 0</math>, then we divide by it, yielding
 +
 +
<cmath>e^{5i\theta} = e^{-i\theta}.</cmath>
 +
 +
Multiplying by <math>e^{i\theta}</math> cancels the righthand side's exponents:
 +
 +
<cmath>e^{6i\theta} = 1.</cmath>
 +
 +
Hence, <math>z^6 = 1</math> and each of the <math>6</math>th roots of unity satisfies this case.
 +
 +
In total, there are <math>1 + 6 = \boxed{\textbf{(E)} 7}</math> numbers that work.
 +
 +
-Benedict T (countmath 1)
  
 
==Video Solution by OmegaLearn==
 
==Video Solution by OmegaLearn==

Revision as of 12:07, 11 November 2023

Problem

How many complex numbers satisfy the equation $z^5=\overline{z}$, where $\overline{z}$ is the conjugate of the complex number $z$?

$\textbf{(A)} ~2\qquad\textbf{(B)} ~3\qquad\textbf{(C)} ~5\qquad\textbf{(D)} ~6\qquad\textbf{(E)} ~7$

Solution 1

When $z^5=\overline{z}$, there are two conditions: either $z=0$ or $z\neq 0$. When $z\neq 0$, since $z^5=\overline{z}$, $|z|=1$. $z^5\cdot z=z^6=\overline{z}\cdot z=|z|^2=1$. Consider the $r(\cos \theta +i\sin \theta)$ form, when $z^6=1$, there are 6 different solutions for $z$. Therefore, the number of complex numbers satisfying $z^5=\bar{z}$ is $\boxed{\textbf{(E)} 7}$.

~plasta

Solution 2

Let $z = re^{i\theta}.$ We now have $\overline{z} = re^{-i\theta},$ and want to solve

\[re^{5i\theta} = re^{-i\theta}.\]

From this, we have $r = 0$ as a solution, which gives $z = 0$. If $r\neq 0$, then we divide by it, yielding

\[e^{5i\theta} = e^{-i\theta}.\]

Multiplying by $e^{i\theta}$ cancels the righthand side's exponents:

\[e^{6i\theta} = 1.\]

Hence, $z^6 = 1$ and each of the $6$th roots of unity satisfies this case.

In total, there are $1 + 6 = \boxed{\textbf{(E)} 7}$ numbers that work.

-Benedict T (countmath 1)

Video Solution by OmegaLearn

https://youtu.be/rbdIrmOyczk

Video Solution

https://youtu.be/m627Mjp3PkM

~Steven Chen (Professor Chen Education Palace, www.professorchenedu.com)

See also

2023 AMC 12A (ProblemsAnswer KeyResources)
Preceded by
Problem 13
Followed by
Problem 15
1 2 3 4 5 6 7 8 9 10 11 12 13 14 15 16 17 18 19 20 21 22 23 24 25
All AMC 12 Problems and Solutions

The problems on this page are copyrighted by the Mathematical Association of America's American Mathematics Competitions. AMC logo.png